Integer carries Factorial

Find the sum of all distinct possible last digits of 2 a ! + 3 b ! \large 2^{a!} + 3^{b!} , where a a and b b are non-negative integers.


The answer is 25.

This section requires Javascript.
You are seeing this because something didn't load right. We suggest you, (a) try refreshing the page, (b) enabling javascript if it is disabled on your browser and, finally, (c) loading the non-javascript version of this page . We're sorry about the hassle.

1 solution

Md Zuhair
Feb 15, 2017

Relevant wiki: General Diophantine Equations - Problem Solving

Here 2 a ! + 3 b ! = r ( m o d 10 ) 2^{a!} + 3^{b!} = r (mod 10)

Now For b 2 \leq 2 , We can have

2 a ! 2^{a!} can end with the numbers in the set {2,4,8,6} Now 3 b ! 3^{b!} for b 2 \leq 2 We have set of values as {3,9}

Hence 2 a ! + 3 b ! = 5 , 7 , 1 , 9 , 3 m o d ( 10 ) 2^{a!} + 3^{b!} = 5 , 7, 1 ,9 , 3 mod(10) ------ Set A

Again for b > 2 b > 2 We have 3 2 = 1 m o d 10 3^{2} = -1 mod 10

So For 3 6 o r 3 b ! 3^6 or 3^{b!} for b > 0 b>0 we have number ending with {1 , 9}

Hence with 2 a ! + 3 b ! = 3 , 5 , 9 , 7 , 1 ( m o d 10 ) 2^{a!} + 3^{b!} = 3, 5, 9, 7 , 1 (mod 10) ------- Set B

Hence Our values of r = A \cup B == {1,3,5,7,9}

So Sum of all r’s = {1+3+5+7+9} = 25 \boxed{25}

As the number given is an odd number always that it may ends ends with 1.3.5.7.9 1.3.5.7.9 .But we have to show that the expression covers all units digit as (\1,3,5,7,9).You showed that

Kushal Bose - 4 years, 3 months ago

Log in to reply

Yes... so its correct?

Md Zuhair - 4 years, 3 months ago

Log in to reply

Can u post it by using latex ?

Kushal Bose - 4 years, 3 months ago

Log in to reply

@Kushal Bose Actualy using phone now.. When I will be back on computer .. That is next day.. I will post it. But isnt it correct?

Md Zuhair - 4 years, 3 months ago

Log in to reply

@Md Zuhair Your approach is correct but make it clear .

Kushal Bose - 4 years, 3 months ago

Log in to reply

@Kushal Bose Okay.. I will try in Latex

Md Zuhair - 4 years, 3 months ago

@Md Zuhair -Actually 2 a ! ≢ 8 ( m o d 10 ) 2^{a!}\not\equiv8\pmod{10} for any a a

2 a ! 8 ( m o d 10 ) 2^{a!}\equiv 8\pmod{10} iff a ! 3 ( m o d 4 ) a!\equiv3\pmod{4} which is not true for any a

for, a = 0 , 1 a=0,1 , 2 a ! 2 ( m o d 10 ) 2^{a!}\equiv 2\pmod{10}

a = 2 , 3 a=2,3 , 2 a ! 4 ( m o d 10 ) 2^{a!}\equiv 4\pmod{10}

a 4 , 2 a ! 6 ( m o d 10 ) a\geq4 ,2^{a!}\equiv 6\pmod{10}

Anirudh Sreekumar - 4 years, 2 months ago

0 pending reports

×

Problem Loading...

Note Loading...

Set Loading...